25
MATHS – JEE MAINS – SHIFT – I 1. The sum of coefficients of even powers of x in [latex]{{\left( x+\ sqrt{{{x}^{3}}-1} \right)}^{6}}+{{\left( x-\sqrt{{{x}^{3}}-1} \ right)}^{6}}[/latex] (A) 23 (B) 24 (C) 18 (D) 21 Solution: [latex]{{\left( x+y \right)}^{n}}+{{\left( x-y \right)}^{n}}=2\ left( n{{c}_{0}}{{x}^{n}}+n{{c}_{2}}{{x}^{n-2}}{{y}^{2}}+n{{c}_{4}}{{x}^{n-4}}. {{y}^{4}}+.... \right)[/latex] [latex]{{\left( x+\sqrt{{{x}^{3}}-1} \right)}^{6}}+{{\left( x-\ sqrt{{{x}^{3}}-1} \right)}^{6}}=2\left[ 6{{c}_{0}}{{x}^{6}}+6{{c}_{2}}. {{x}^{4}}\left( {{x}^{3}}-1 \right)+6{{c}_{4}}{{x}^{2}}{{\left( {{x}^{3}}-1 \ right)}^{2}}+6{{c}_{6}}{{\left( {{x}^{3}}-1 \right)}^{3}} \right][/latex] [latex]=2\left[ {{x}^{6}}+15{{x}^{4}}\ left( {{x}^{3}}-1 \right)+15{{x}^{2}}\left( {{x}^{6}}-2{{x}^{3}}+1 \right)+{{\ left( {{x}^{3}}-1 \right)}^{3}} \right][/latex] [latex]=2\left[ {{x}^{6}}+15{{x}^{7}}-15{{x}^{4}} +15{{x}^{8}}-30{{x}^{5}}+15{{x}^{2}}+{{x}^{9}}-3{{x}^{6}}+3\left( {{x}^{3}}-1 \ right) \right][/latex] Terms with even power of [latex]x=2\left[ {{x}^{6}}-15{{x}^{4}} +15{{x}^{8}}+15{{x}^{2}}-3{{x}^{6}}-1 \right][/latex] [latex]\therefore[/latex] Sum of coefficients [latex]=2\left[ 1-\ cancel{15}+\cancel{15}+15-3-1 \right]=24[/latex] 2. Let [latex]\sin \left( \alpha -\beta \right)=\frac{5}{13}[/latex]and [latex]\cos \left( \alpha +\beta \right)=\frac{3}{5}[/latex]where [latex]\ 1 BYJU’s Classes 4 th Floor, Prince Kushal Towers, Mount Road, Chennai -02. PH: 9289000333

cdn1.byjus.com · Web view[latex]A=\left[ \begin{matrix} \cos \alpha & -\sin \alpha \\ \sin \alpha & \cos \alpha \\ \end{matrix} \right]\Rightarrow {{A}^{2}}\left[ \begin{matrix}

  • Upload
    others

  • View
    1

  • Download
    0

Embed Size (px)

Citation preview

Page 1: cdn1.byjus.com · Web view[latex]A=\left[ \begin{matrix} \cos \alpha & -\sin \alpha \\ \sin \alpha & \cos \alpha \\ \end{matrix} \right]\Rightarrow {{A}^{2}}\left[ \begin{matrix}

MATHS – JEE MAINS – SHIFT – I

1. The sum of coefficients of even powers of x in [latex]{{\left( x+\sqrt{{{x}^{3}}-1} \right)}^{6}}+{{\left( x-\

sqrt{{{x}^{3}}-1} \right)}^{6}}[/latex]

(A) 23 (B) 24 (C) 18 (D) 21

Solution:

[latex]{{\left( x+y \right)}^{n}}+{{\left( x-y \right)}^{n}}=2\left( n{{c}_{0}}{{x}^{n}}+n{{c}_{2}}

{{x}^{n-2}}{{y}^{2}}+n{{c}_{4}}{{x}^{n-4}}.{{y}^{4}}+.... \right)[/latex]

[latex]{{\left( x+\sqrt{{{x}^{3}}-1} \right)}^{6}}+{{\left( x-\sqrt{{{x}^{3}}-1} \right)}^{6}}=2\

left[ 6{{c}_{0}}{{x}^{6}}+6{{c}_{2}}.{{x}^{4}}\left( {{x}^{3}}-1 \right)+6{{c}_{4}}{{x}^{2}}{{\

left( {{x}^{3}}-1 \right)}^{2}}+6{{c}_{6}}{{\left( {{x}^{3}}-1 \right)}^{3}} \right][/latex]

[latex]=2\left[ {{x}^{6}}+15{{x}^{4}}\left( {{x}^{3}}-1 \right)

+15{{x}^{2}}\left( {{x}^{6}}-2{{x}^{3}}+1 \right)+{{\left( {{x}^{3}}-1 \right)}^{3}} \right][/latex]

[latex]=2\left[ {{x}^{6}}+15{{x}^{7}}-15{{x}^{4}}+15{{x}^{8}}-

30{{x}^{5}}+15{{x}^{2}}+{{x}^{9}}-3{{x}^{6}}+3\left( {{x}^{3}}-1 \right) \right][/latex]

Terms with even power of [latex]x=2\left[ {{x}^{6}}-15{{x}^{4}}+15{{x}^{8}}+15{{x}^{2}}-3{{x}^{6}}-

1 \right][/latex]

[latex]\therefore[/latex] Sum of coefficients [latex]=2\left[ 1-\cancel{15}+\cancel{15}+15-3-1

\right]=24[/latex]

2. Let [latex]\sin \left( \alpha -\beta \right)=\frac{5}{13}[/latex]and [latex]\cos \left( \alpha +\beta \right)=\frac{3}

{5}[/latex]where [latex]\alpha ,\beta \in \left( 0,\frac{\pi }{4} \right)[/latex] then [latex]Tan2\alpha =[/latex]

(A) [latex]\frac{63}{16}[/latex] (B) [latex]\frac{61}{16}[/latex] (C) [latex]\

frac{65}{16}[/latex] (D) [latex]\frac{32}{9}[/latex]

Solution:

[latex]\sin \left( \alpha -\beta \right)=\frac{5}{13}\Rightarrow \cos \left( \alpha -\beta \right)=\frac{12}{13}\

Rightarrow Tan\left( \alpha -\beta \right)=\frac{5}{12}[/latex]

[latex]\cos \left( \alpha +\beta \right)=\frac{3}{5}\Rightarrow \sin \left( \alpha +\beta \right)=\frac{4}{5}\

Rightarrow Tan\left( \alpha +\beta \right)=\frac{4}{3}[/latex]

[latex]Tan2\alpha =Tan\left[ \left( \alpha +\beta \right)+\left( \alpha -\beta \right) \right][/latex]

1 BYJU’s Classes4th Floor, Prince Kushal Towers, Mount Road, Chennai -02. PH: 9289000333

Page 2: cdn1.byjus.com · Web view[latex]A=\left[ \begin{matrix} \cos \alpha & -\sin \alpha \\ \sin \alpha & \cos \alpha \\ \end{matrix} \right]\Rightarrow {{A}^{2}}\left[ \begin{matrix}

MATHS – JEE MAINS – SHIFT – I

[latex]=\frac{Tan\left( \alpha +\beta \right)+Tan\left( \alpha -\beta \right)}{1-Tan\left( \alpha +\beta \

right).\,Tan\left( \alpha -\beta \right)}=\frac{\frac{4}{3}+\frac{5}{12}}{1-\frac{4}{3}.\frac{5}{12}}=\frac{16+5}{\

frac{36-20}{3}}=\frac{63}{16}[/latex]

3. The line [latex]x+y=n,\,\,n\in N[/latex] makes intercepts with [latex]{{x}^{2}}+{{y}^{2}}=16.[/latex] Then

the sum of squares of all possible intercepts

(A) [latex]\frac{105}{4}[/latex] (B) 105 (C) 210 (D) [latex]\frac{105}

{2}[/latex]

Solution:

[latex]{{x}^{2}}+{{y}^{2}}=16[/latex]

Centre = (0, 0) OA = P

Radius = 4

P [latex]=\frac{n}{\sqrt{2}}[/latex]

To make the intercepts

[latex]\frac{n}{\sqrt{2}}<4\Rightarrow n<4\

sqrt{2}[/latex]

Length of intercepts [latex]=\sqrt{{{r}^{2}}-{{p}^{2}}}=\sqrt{16-\frac{{{n}^{2}}}{2}}[/latex]

Square of intercepts [latex]=16-\frac{{{n}^{2}}}{2},n\in N[/latex]

Sum of squares of intercepts

[latex]=\left( 16-\frac{1}{2} \right)+\left( 16-\frac{4}{2} \right)+\left( 16-\frac{9}{2} \right)+\

left( 16-\frac{16}{2} \right)+\left( 16-\frac{25}{2} \right)[/latex]

[latex]=80-\frac{55}{2}=\frac{105}{2}[/latex]

4. [latex]\int{\frac{\sin \frac{5x}{2}}{\sin \frac{x}{2}}}.\,dx=[/latex]

(A) [latex]x+2\sin x+\sin 2x+c[/latex] (B) [latex]x+2\cos x+\sin 2x+c[/latex]

(C) [latex]x-2\sin x+\sin 2x+c[/latex] (D) [latex]x+2\sin x-\sin 2x+c[/latex]

Solution:

2 BYJU’s Classes4th Floor, Prince Kushal Towers, Mount Road, Chennai -02. PH: 9289000333

Page 3: cdn1.byjus.com · Web view[latex]A=\left[ \begin{matrix} \cos \alpha & -\sin \alpha \\ \sin \alpha & \cos \alpha \\ \end{matrix} \right]\Rightarrow {{A}^{2}}\left[ \begin{matrix}

MATHS – JEE MAINS – SHIFT – I

[latex]\int{\frac{\sin \frac{5x}{2}}{\sin \frac{x}{2}}}.\,dx=\int{\frac{2\sin \frac{5x}{2}.\cos \frac{x}{2}}{2\

sin \frac{x}{2}.\cos \frac{x}{2}}}.\,dx=\int{\frac{\sin 3x+\sin 2x}{\sin 2\left( \frac{x}{2} \right)}}.\,dx[/latex]

[latex]=\int{\frac{\sin 3x}{\sin x}}.\,dx+\int{\frac{\sin 2x}{\sin x}}.\,dx[/latex]

[latex]=\int{\frac{3\sin x-4{{\sin }^{3}}x}{\sin x}}.\,dx+\int{\frac{2\sin x\cos x}{\sin

x}}dx[/latex]

[latex]=3\int{dx-4\int{{{\sin }^{2}}x.\,dx}}+2\int{\cos x\,dx=3x-4}\int{\frac{1-\cos 2x}

{2}}.\,dx+2\sin x[/latex]

[latex]=3x-2x+\sin 2x+2\sin x+c=x+2\sin x+\sin 2x+c[/latex]

5. The area bounded by the curve [latex]y\le {{x}^{2}}+3x,0\le y\le 4,0\le x\le 3[/latex] is

(A) [latex]\frac{59}{6}[/latex] (B) [latex]\frac{57}{4}[/latex] (C) [latex]\

frac{59}{3}[/latex] (D) [latex]\frac{57}{6}[/latex]

Solution:

[latex]y={{x}^{2}}+3x[/latex]

[latex]y=4[/latex]

[latex]\Rightarrow {{x}^{2}}+3x=4[/latex]

[latex]\Rightarrow {{x}^{2}}+3x-4=0\Rightarrow

x=1\,\,or\,\,x=-4[/latex]

Area [latex]=\int\limits_{0}^{1}{\left( {{x}^{2}}+3x \

right).\,dx+}[/latex] Area + Rectangle

[latex]=\left[ \frac{{{x}^{3}}}{3}+\

frac{3{{x}^{2}}}{2} \right]_{0}^{1}+2\left( 4 \right)[/latex]

[latex]=\frac{1}{3}+\frac{3}{2}+8=\frac{59}{6}[/latex]

6. “If you are born in India then you are citizen of India” contrapositive of this statement is

(A) If you are born in India then you are not citizen of India

(B) If you are not citizen of India then you are not born in India

(C) If you are citizen of India then you are not born in India

(D) If you are citizen of India then you are born in India

3 BYJU’s Classes4th Floor, Prince Kushal Towers, Mount Road, Chennai -02. PH: 9289000333

Page 4: cdn1.byjus.com · Web view[latex]A=\left[ \begin{matrix} \cos \alpha & -\sin \alpha \\ \sin \alpha & \cos \alpha \\ \end{matrix} \right]\Rightarrow {{A}^{2}}\left[ \begin{matrix}

MATHS – JEE MAINS – SHIFT – I

Solution:

Statement: [latex]p\Rightarrow q[/latex]

Contrapositive: [latex]\tilde{\ }p\Rightarrow \,\tilde{\ }q[/latex]

[latex]\therefore[/latex] Answer is if you are not citizen of India then you are not born in India.

7. [latex]A=\left[ \begin{matrix} \cos \alpha & -\sin \alpha \\ \sin \alpha & \cos \alpha \\ \end{matrix}

\right][/latex] and [latex]{{A}^{32}}=\left[ \begin{matrix} 0 & -1 \\ 1 & 0 \\ \end{matrix} \right][/latex]. then

α may be

(A) 0 (B) [latex]\frac{\pi }{32}[/latex] (C) [latex]\frac{\pi

}{64}[/latex] (D) [latex]\frac{\pi }{16}[/latex]

Solution:

[latex]A=\left[ \begin{matrix} \cos \alpha & -\sin \alpha \\ \sin \alpha & \cos \alpha \\ \end{matrix} \right]\

Rightarrow {{A}^{2}}\left[ \begin{matrix} {{\cos }^{2}}\alpha -{{\sin }^{2}}\alpha & -\cos \alpha \sin \alpha -\cos \

alpha \sin \alpha \\ \sin \alpha \cos \alpha +\sin \alpha \cos \alpha & -{{\sin }^{2}}\alpha +{{\cos }^{2}}\alpha \\ \

end{matrix} \right][/latex]

[latex]\Rightarrow {{A}^{2}}=\left[ \begin{matrix} \cos 2\alpha & -\sin 2\alpha \\ \sin

2\alpha & \cos 2\alpha \\ \end{matrix} \right][/latex]

[latex]\Rightarrow {{A}^{n}}=\left[ \begin{matrix} \cos \,n\alpha & -\sin \,n\alpha \\ \

sin \,n\alpha & \cos \,n\alpha \\ \end{matrix} \right][/latex]

[latex]\Rightarrow \cos 32\alpha =0[/latex] and [latex]\sin 32\alpha =1[/latex]

[latex]\Rightarrow 32\alpha =2n\pi +\frac{\pi }{2}[/latex]

[latex]\alpha =\frac{n\pi }{16}+\frac{\pi }{64},\,n\in I[/latex]

8. Shortest distance between the curves [latex]{{y}^{2}}=x-2[/latex] and [latex]y=x[/latex] is

(A) greater than 4 (B) less than 2 (C) greater than 3 (D) greater than 2

Solution:

[latex]{{y}^{2}}=x-2[/latex]

4 BYJU’s Classes4th Floor, Prince Kushal Towers, Mount Road, Chennai -02. PH: 9289000333

Page 5: cdn1.byjus.com · Web view[latex]A=\left[ \begin{matrix} \cos \alpha & -\sin \alpha \\ \sin \alpha & \cos \alpha \\ \end{matrix} \right]\Rightarrow {{A}^{2}}\left[ \begin{matrix}

MATHS – JEE MAINS – SHIFT – I

[latex]2y.\frac{dy}{dx}=1[/latex]

[latex]\frac{dy}{dx}=\frac{1}{2y}[/latex]

Slope of the line [latex]y=x[/latex] is 1

Slope of the tangent to the curve [latex]=\

frac{1}{2y}[/latex]

[latex]\frac{1}{2y}=1\Rightarrow

y=\frac{1}{2}\Rightarrow K=\frac{1}{2}[/latex]

[latex]\therefore \,\,N=\

left( \frac{9}{4},\frac{1}{2} \right)[/latex]

Minimum distance = MN = perpendicular

distance from point N to line [latex]y=x[/latex]

[latex]\left| \frac{a{{x}_{1}}+b{{y}_{1}}+c}{\sqrt{{{a}^{2}}+{{b}^{2}}}} \right|=\left| \

frac{1\left( \frac{9}{4} \right)-1\left( \frac{1}{2} \right)}{\sqrt{1+1}} \right|=\frac{7}{4\sqrt{2}}[/latex] which is less

than 2

9. [latex]\underset{x\to 0}{\mathop{\lim }}\,\frac{\sin 2x}{\sqrt{2}-\sqrt{1+\cos x}}=[/latex]

(A) [latex]\sqrt{2}[/latex] (B) 2 (C) 4 (D) [latex]4\

sqrt{2}[/latex]

Solution:

Rationalise the denominater

[latex]\underset{x\to 0}{\mathop{\lim }}\,\frac{{{\sin }^{2}}x\left[ \sqrt{2}+\sqrt{1+\cos x} \right]}{2-\

left( 1+\cos x \right)}=\underset{x\to 0}{\mathop{\lim }}\,\frac{{{\sin }^{2}}x\left( \sqrt{2}+\sqrt{1+\cos x} \right)}

{1-\cos x}[/latex]

[latex]=\underset{x\to 0}{\mathop{Lt}}\,\frac{{{\sin }^{2}}x\left( \sqrt{2}+\sqrt{1+\cos x} \right)}{2{{\

sin }^{2}}\frac{x}{2}}=\underset{x\to 0}{\mathop{Lt}}\,\frac{{{\sin }^{2}}x}{{{x}^{2}}}.\frac{1}{2}\frac{1}{{{\

left( \frac{\sin \frac{x}{2}}{\frac{x}{2}} \right)}^{2}}\times \frac{1}{4}}\left( \sqrt{2}+\sqrt{1+\cos } \right)[/latex]

[latex]=\frac{4}{2}\left( \sqrt{2}+\sqrt{1+1} \right)=4\sqrt{2}[/latex]

5 BYJU’s Classes4th Floor, Prince Kushal Towers, Mount Road, Chennai -02. PH: 9289000333

Page 6: cdn1.byjus.com · Web view[latex]A=\left[ \begin{matrix} \cos \alpha & -\sin \alpha \\ \sin \alpha & \cos \alpha \\ \end{matrix} \right]\Rightarrow {{A}^{2}}\left[ \begin{matrix}

MATHS – JEE MAINS – SHIFT – I

10. How many 9 digit numbers can be formed by using the digits 1, 1, 2, 2, 2, 2, 3, 4, 4 such that odd numbers

occur at even places.

(A) 160 (B) 175 (C) 180 (D) 220

Solution:

Even places = 4

Odd numbers = 3

Even numbers = 6

[latex]\therefore[/latex] Number of numbers [latex]=4{{c}_{3}}\times \frac{3!}{2!}\times \frac{6!}{4!

2!}=180[/latex]

11. Let [latex]g\left( x \right)=\ell n\left( x \right)[/latex] and [latex]f\left( x \right)=\left( \frac{1-x\cos x}{1+x\cos

x} \right)[/latex] then [latex]\int\limits_{-\frac{\pi }{4}}^{\frac{\pi }{4}}{g\left[ f\left( x \right) \

right]}.\,dx=[/latex]

(A) [latex]\ell n\,1[/latex] (B) [latex]\ell n\,2[/latex] (C) [latex]\ell

n\,e[/latex] (D) [latex]\ell n\,4[/latex]

Solution:

[latex]g\left[ f\left( x \right) \right]=g\left[ \frac{1-x\cos x}{1+x\cos x} \right]=\ell n\left( \frac{1-x\cos x}{1+x\

cos x} \right).\,dx[/latex]

[latex]I=\int\limits_{-\frac{\pi }{4}}^{\frac{\pi }{4}}{g\left[ f\left( x \right) \right]dx=\int\limits_{-\frac{\pi }

{4}}^{\frac{\pi }{4}}{\ell n\left( \frac{1-x\cos x}{1+x\cos x} \right)}\,dx}[/latex] … (1)

[latex]\int\limits_{a}^{b}{f\left( x \right)}=\int\limits_{a}^{b}{f\left( a+b-x \right)}[/latex]

[latex]I=\int\limits_{-\frac{\pi }{4}}^{\frac{\pi }{4}}{\ell n\left( \frac{1+x\cos x}{1-x\cos x} \right)}[/latex]

… (2)

Add (1) and (2)

[latex]2I=\int\limits_{-\frac{\pi }{4}}^{\frac{\pi }{4}}{\left[ \ell n\left( \frac{1+x\cos x}{1-x\cos x} \right)+\

left( \frac{1-x\cos x}{1+x\cos x} \right) \right]}\,dx[/latex]

[latex]=\int\limits_{-\frac{\pi }{4}}^{\frac{\pi }{4}}{\ell n}\left( \frac{1+x\cos x}{1-x\cos x}.\frac{1-x\cos x}

{1+x\cos x} \right)\,dx[/latex] [latex]\left[ \because \,\,\log ab=\log a+\log b \right][/latex]

6 BYJU’s Classes4th Floor, Prince Kushal Towers, Mount Road, Chennai -02. PH: 9289000333

Page 7: cdn1.byjus.com · Web view[latex]A=\left[ \begin{matrix} \cos \alpha & -\sin \alpha \\ \sin \alpha & \cos \alpha \\ \end{matrix} \right]\Rightarrow {{A}^{2}}\left[ \begin{matrix}

MATHS – JEE MAINS – SHIFT – I

[latex]2I=\int\limits_{-\frac{\pi }{4}}^{\frac{\pi }{4}}{\ell n}\,1=0\Rightarrow I=0[/latex]

12. Let [latex]2.20{{C}_{0}}+5.20{{C}_{1}}+8.20{{C}_{2}}+...+62.20{{C}_{20}}=[/latex]

(A) [latex]{{16.2}^{22}}[/latex] (B) [latex]{{8.2}^{20}}[/latex] (C) [latex]

{{8.2}^{21}}[/latex] (D) [latex]{{16.2}^{21}}[/latex]

Solution:

[latex]2.20{{C}_{0}}+5.20{{C}_{1}}+8.20{{C}_{2}}+...+62.20{{C}_{20}}[/latex]

[latex]=\sum\limits_{r=0}^{20}{\left( 3r+2 \right)20{{C}_{r}}}[/latex]

[latex]=3\sum\limits_{r=0}^{20}{r.\,20{{C}_{r}}}+2.\sum\limits_{r=0}^{20}{20{{C}_{r}}}.[/latex]

[latex]=3.20\sum\limits_{r=1}^{20}{19{{C}_{r-1}}+2\left( 20{{C}_{0}}+20{{C}_{1}}+...

+20{{C}_{20}} \right)}[/latex]

[latex]={{60.2}^{19}}+{{2.2}^{20}}={{2}^{21}}\left( 15+1 \right)={{2}^{21}}.16.[/latex]

13. Sum of natural numbers between 100 and 200 whose HCF with 91 should be more than 1.

(A) 1121 (B) 3210 (C) 3121 (D) 1520

Solution:

Given numbers: 101, 102, 103, … 198, 199

[latex]91=13\times 7[/latex]

HCF of 91 and a number is more than 1 means the number should be either multiple of 7 or 13.

[latex]\therefore[/latex] Sum of the numbers = (Numbers divisible by 7) + (Numbers divisible by 13) –

(Numbers divisible by 91)

[latex]=\left( 105+112+...+196 \right)+\left( 104+117+...195 \right)-\left( 182

\right)[/latex]

[latex]=\frac{14}{2}\left[ 105+196 \right]+\frac{8}{2}\left[ 104+195

\right]-182[/latex]

[latex]=7\left( 301 \right)+4\left( 299 \right)-182=2107+1196-182[/latex]

[latex]=3121[/latex]

7 BYJU’s Classes4th Floor, Prince Kushal Towers, Mount Road, Chennai -02. PH: 9289000333

Page 8: cdn1.byjus.com · Web view[latex]A=\left[ \begin{matrix} \cos \alpha & -\sin \alpha \\ \sin \alpha & \cos \alpha \\ \end{matrix} \right]\Rightarrow {{A}^{2}}\left[ \begin{matrix}

MATHS – JEE MAINS – SHIFT – I

14. If mean and variance of 7 variates are 8 and 16 respectively and five of them are 2, 4, 10, 12, 14 then the

product of remaining two variates is

(A) 49 (B) 48 (C) 45 (D) 40

Solution:

Let the remaining two variates be x and y

Given mean [latex]=7\Rightarrow \frac{x+y+2+4+10+12+14}{7}=8[/latex]

[latex]\Rightarrow x+y=14[/latex] … (1)

And variance [latex]=16\Rightarrow \frac{{{x}^{2}}+{{y}^{2}}+4+16+100+144+196}{7}-{{\left( 8 \

right)}^{2}}=16[/latex]

[latex]\Rightarrow {{x}^{2}}+{{y}^{2}}=100[/latex] … (2)

From (1) [latex]x+y=14\Rightarrow {{\left( x+y \right)}^{2}}={{\left( 14 \right)}^{2}}[/latex]

[latex]\Rightarrow {{x}^{2}}+{{y}^{2}}+2xy=196[/latex]

[latex]\Rightarrow 2xy=196-100\Rightarrow xy=48[/latex]

15. If α and β are the roots of [latex]{{x}^{2}}-2x+2=0[/latex] then the minimum value of n such that [latex]{{\

left( \frac{\alpha }{\beta } \right)}^{n}}=1[/latex]

(A) 4 (B) 3 (C) 2 (D) 5

Solution:

[latex]{{x}^{2}}-2x+2=0\Rightarrow x=\frac{2\pm \sqrt{4-8}}{2}\Rightarrow x=1\pm i[/latex]

[latex]\alpha =1+i[/latex] and [latex]\beta =1-i[/latex]

[latex]\frac{\alpha }{\beta }=\frac{1+i}{1-i}=\frac{1+i}{1-i}\times

\frac{1+i}{1+i}=\frac{2i}{2}=i[/latex]

[latex]{{\left( \frac{\alpha }{\beta } \right)}^{n}}={{i}^{n}}[/latex]

Given [latex]{{i}^{n}}=1\Rightarrow n=4,8,12,....\,\,\,n\in N[/latex]

Minimum value of [latex]n=4[/latex]

8 BYJU’s Classes4th Floor, Prince Kushal Towers, Mount Road, Chennai -02. PH: 9289000333

Page 9: cdn1.byjus.com · Web view[latex]A=\left[ \begin{matrix} \cos \alpha & -\sin \alpha \\ \sin \alpha & \cos \alpha \\ \end{matrix} \right]\Rightarrow {{A}^{2}}\left[ \begin{matrix}

MATHS – JEE MAINS – SHIFT – I

16. Solution of differential equation [latex]{{\left( {{x}^{2}}+1 \right)}^{2}}\frac{dy}{dx}=2x\left( {{x}^{2}}+1

\right)y=1[/latex]is

(A) [latex]y=\frac{Ta{{n}^{-1}}x}{{{x}^{2}}+1}+c[/latex] (B) [latex]y=Ta{{n}^{-1}}x+c[/latex] (C)

[latex]y\left( {{x}^{2}}+1 \right)=Ta{{n}^{-1}}x+c[/latex] (D) [latex]y\left( Ta{{n}^{-1}}x \

right)={{x}^{2}}+c[/latex]

Solution:

[latex]{{\left( {{x}^{2}}+1 \right)}^{2}}\frac{dy}{dx}+2x\left( {{x}^{2}}+1 \right)y=1[/latex]

[latex]\frac{dy}{dx}+\frac{2x}{{{x}^{2}}+1}.\,y=\frac{1}{{{\left( {{x}^{2}}+1

\right)}^{2}}}[/latex]

It is linear differential equation where [latex]P=\frac{2x}{{{x}^{2}}+1},Q=\frac{1}{{{\left( {{x}^{2}}+1 \

right)}^{2}}}[/latex]

If [latex]={{e}^{\int{P.\,dx}}}={{e}^{\int{\frac{2x}{{{x}^{2}}+1}.\,dx}}}={{e}^{\ell n\left( {{x}^{2}}+1 \

right)}}={{x}^{2}}+1[/latex]

[latex]y.\,IF=\int{Q.\,IF.\,dx}[/latex]

[latex]y\left( {{x}^{2}}+1 \right)=\int{\frac{1}{{{\left( {{x}^{2}}+1 \right)}^{2}}}}.\left( {{x}^{2}}

+1 \right)dx[/latex]

[latex]y\left( {{x}^{2}}+1 \right)=\int{\frac{1}{{{x}^{2}}+1}}.\,dx[/latex]

[latex]y\left( {{x}^{2}}+1 \right)=Ta{{n}^{-1}}x+c[/latex]

17. If [latex]f\left( x \right)=\log \left( \frac{1-x}{1+x} \right)[/latex] then [latex]f\left( \frac{2x}{1+{{x}^{2}}} \

right)[/latex] is equal to

(A) [latex]f\left( x \right)[/latex] (B) [latex]2f\left( x \right)[/latex] (C) [latex]-2f\left( x \

right)[/latex] (D) [latex]{{\left[ f\left( x \right) \right]}^{2}}[/latex]

Solution:

[latex]f\left( x \right)=\log \left( \frac{1-x}{1+x} \right)[/latex]

[latex]f\left( \frac{2x}{1+{{x}^{2}}} \right)=\log \left[ \frac{1-\frac{2x}{1+{{x}^{2}}}}{1+\frac{2x}{1+

{{x}^{2}}}} \right]=\log \left( \frac{1+{{x}^{2}}-2x}{1+{{x}^{2}}+2x} \right)=\log {{\left( \frac{1-x}{1+x} \

right)}^{2}}[/latex]

9 BYJU’s Classes4th Floor, Prince Kushal Towers, Mount Road, Chennai -02. PH: 9289000333

Page 10: cdn1.byjus.com · Web view[latex]A=\left[ \begin{matrix} \cos \alpha & -\sin \alpha \\ \sin \alpha & \cos \alpha \\ \end{matrix} \right]\Rightarrow {{A}^{2}}\left[ \begin{matrix}

MATHS – JEE MAINS – SHIFT – I

[latex]=2.log\left( \frac{1-x}{1+x} \right)[/latex]

[latex]=2.\,f\left( x \right)[/latex]

18. Given that [latex]A\subset B[/latex] then identify the correct statement.

(A) [latex]P\left( \frac{A}{B} \right)=P\left( A \right)[/latex] (B) [latex]P\left( \frac{A}{B} \right)\le P\left(

A \right)[/latex] (C) [latex]P\left( \frac{A}{B} \right)\ge P\left( A \right)[/latex] (D) [latex]P\left( \frac{A}{B}

\right)=P\left( A \right)-P\left( B \right)[/latex]

Solution:

[latex]P\left( \frac{A}{B} \right)=\frac{P\left( A\cap B \right)}{P\left( B \right)}=\frac{P\left( A \right)}{P\

left( B \right)}\ge P\left( A \right)[/latex]

19. Find the value of ‘c’ for which the following equations have non-trivial solutions.

[latex]cx-y-z=0,-cx+y-cz=0,x+y-cz=0[/latex]

(A) [latex]\frac{1}{2}[/latex] (B) [latex]-1[/latex] (C) 2

(D) 0

Solution:

[latex]\left| \begin{matrix} c & -1 & -1 \\ -c & 1 & -c \\ 1 & 1 & -c \\ \end{matrix} \right|=0[/latex]

[latex]\Rightarrow c\left( -c+c \right)+1\left( {{c}^{2}}+c \right)-1\left( -c-1 \right)=0[/latex]

[latex]\Rightarrow {{c}^{2}}+2c+1=0\Rightarrow {{\left( c+1 \right)}^{2}}=0\Rightarrow c=-1[/latex]

20. Let [latex]2y={{\left[ {{\cot }^{-1}}\left( \frac{\sqrt{3}\cos x+\sin x}{\cos x-\sqrt{3}\sin x} \right) \

right]}^{2}}[/latex] then [latex]\frac{dy}{dx}=[/latex]

(A) [latex]x-\frac{\pi }{6}[/latex] (B) [latex]x+\frac{\pi }{6}[/latex] (C)

[latex]2x-\frac{\pi }{6}[/latex] (D) [latex]2x-\frac{\pi }{3}[/latex]

Solution:

[latex]2y={{\left[ {{\cot }^{-1}}\left( \frac{\sqrt{3}\cos x+\sin x}{\cos x-\sqrt{3}\sin x} \right)

\right]}^{2}}[/latex]

[latex]2y={{\left[ {{\cot }^{-1}}\left( \frac{\sqrt{3}+Tan\,x}{1-\sqrt{3}Tan\,x} \right) \right]}^{2}}[/latex]

[[latex]\because[/latex] Divide Nr & Dr with cos x]

10 BYJU’s Classes4th Floor, Prince Kushal Towers, Mount Road, Chennai -02. PH: 9289000333

Page 11: cdn1.byjus.com · Web view[latex]A=\left[ \begin{matrix} \cos \alpha & -\sin \alpha \\ \sin \alpha & \cos \alpha \\ \end{matrix} \right]\Rightarrow {{A}^{2}}\left[ \begin{matrix}

MATHS – JEE MAINS – SHIFT – I

[latex]2y={{\left[ {{\cot }^{-1}}\left( \frac{Tan60+Tan\,x}{1-Tan60.\,Tanx} \right) \right]}^{2}}[/latex]

[latex]2y={{\left[ {{\cot }^{-1}}\left( Tan\frac{\pi }{3}+x \right) \right]}^{2}}[/latex]

[latex]2y{{\left\{ \frac{\pi }{2}-Ta{{n}^{-1}}\left[ Tan\left( \frac{\pi }{3}+x \right) \right]

\right\}}^{2}}[/latex]

[latex]2y={{\left( \frac{\pi }{2}-\frac{\pi }{3}-x \right)}^{2}}\Rightarrow 2y={{\left( \frac{\pi }{6}-x \

right)}^{2}}[/latex]

[latex]2y=\frac{{{\pi }^{2}}}{36}+{{x}^{2}}-\frac{\pi x}{3}[/latex]

[latex]2.\,\frac{dy}{dx}=2x-\frac{\pi }{3}[/latex]

[latex]\frac{dy}{dx}=x-\frac{\pi }{6}[/latex]

21. Let [latex]{{S}_{1}}[/latex]is set of minima and [latex]{{S}_{2}}[/latex]is set of maxima for the curve

[latex]y=9{{x}^{4}}+12{{x}^{3}}-36{{x}^{2}}-25[/latex] then

(A) [latex]{{S}_{1}}=\left\{ -2,-1 \right\},{{S}_{2}}=\left\{ 0 \right\}[/latex] (B) [latex]

{{S}_{1}}=\left\{ -2,1 \right\},{{S}_{2}}=\left\{ 0 \right\}[/latex]

(C) [latex]{{S}_{1}}=\left\{ -2,1 \right\},{{S}_{2}}=\left\{ -1 \right\}[/latex] (D) [latex]

{{S}_{1}}=\left\{ -2,2 \right\},{{S}_{2}}=\left\{ 0 \right\}[/latex]

Solution:

[latex]y=9{{x}^{4}}+12{{x}^{3}}-36{{x}^{2}}-25[/latex]

[latex]\frac{dy}{dx}=36{{x}^{3}}+36{{x}^{2}}-72x[/latex]

[latex]=36x\left( {{x}^{2}}+x-2 \right)[/latex]

[latex]=36x\left( {{x}^{2}}+2x-x-2 \right)[/latex]

[latex]=36x\left( x+2 \right)\left( x-1 \right)[/latex]

Critical points are 0, -2, 1

At [latex]\left\{ -2,1 \right\}\to[/latex] points of minima.

[latex]\left\{ 0 \right\}\to[/latex] point of maxima.

22. Let [latex]{{f}^{11}}\left( x \right)>0[/latex] and [latex]\phi \left( x \right)=f\left( x \right)+f\left( 2-x \right),x\

in \left( 0,2 \right)[/latex] be a function then the function [latex]\phi \left( x \right)[/latex]is 11 BYJU’s Classes4th Floor, Prince Kushal Towers, Mount Road, Chennai -02. PH: 9289000333

Page 12: cdn1.byjus.com · Web view[latex]A=\left[ \begin{matrix} \cos \alpha & -\sin \alpha \\ \sin \alpha & \cos \alpha \\ \end{matrix} \right]\Rightarrow {{A}^{2}}\left[ \begin{matrix}

MATHS – JEE MAINS – SHIFT – I

(A) Increasing in (0, 1) and decreasing in (1, 2) (B) Decreasing in (0, 1) and increasing in (1, 2)

(C) Increasing in (0, 2) (D) Decreasing in (0, 2)

Solution:

Let [latex]y=f\left( x \right),x\in \left( 0,2 \right)[/latex]

[latex]\phi \left( x \right)=f\left( x \right)+f\left( 2-x \right)[/latex]

[latex]\phi '\left( x \right)=f'\left( x \right)+f'\left( 2-x \right)[/latex]

If [latex]\phi '\left( x \right)[/latex] is increasing then [latex]\phi '\left( x \right)>0[/latex]

[latex]\Rightarrow f'\left( x \right)-f'\left( 2-x \right)>0[/latex]

[latex]\Rightarrow f'\left( x \right)>f'\left( 2-x \right)[/latex]

[latex]\Rightarrow x>2-x[/latex] [[latex]f'\left( x \right)[/latex] is

increasing in (0, 2)]

[latex]\Rightarrow x>1[/latex]

[latex]\Rightarrow x\in \left( 1,\,2 \right)[/latex]

It [latex]\phi '\left( x \right)[/latex] is decreasing then [latex]\phi '\left( x \right)<0[/latex]

[latex]\Rightarrow f'\left( x \right)<f'\left( 2-x \right)[/latex]

[latex]\Rightarrow x<1\Rightarrow x\in \left( 0,1 \right)[/latex]

23. Let vertices of the triangle ABC is A(0, 0), B(0, 1) and C(x, y) and perimeter is 4 then locus of ‘C’ is

(A) [latex]9{{x}^{2}}+8{{y}^{2}}+8y=16[/latex] (B) [latex]8{{x}^{2}}+9{{y}^{2}}

+9y=16[/latex]

(C) [latex]9{{x}^{2}}+8{{y}^{2}}-8y=16[/latex] (D) [latex]8{{x}^{2}}+9{{y}^{2}}-

9x=16[/latex]

Solution:

Perimeter = 4

[latex]\sqrt{{{x}^{2}}+{{\left( y-1 \

right)}^{2}}}+\sqrt{{{x}^{2}}+{{y}^{2}}}+1=4[/latex]

12 BYJU’s Classes4th Floor, Prince Kushal Towers, Mount Road, Chennai -02. PH: 9289000333

Page 13: cdn1.byjus.com · Web view[latex]A=\left[ \begin{matrix} \cos \alpha & -\sin \alpha \\ \sin \alpha & \cos \alpha \\ \end{matrix} \right]\Rightarrow {{A}^{2}}\left[ \begin{matrix}

MATHS – JEE MAINS – SHIFT – I

[latex]\sqrt{{{x}^{2}}+{{y}^{2}}-2y+1}=3-\sqrt{{{x}^{2}}+{{y}^{2}}}[/latex]

[latex]\cancel{{{x}^{2}}}+\cancel{{{y}^{2}}}-2y+1=9+\cancel{{{x}^{2}}}+\cancel{{{y}^{2}}}-6\

sqrt{{{x}^{2}}+{{y}^{2}}}[/latex]

[latex]6\sqrt{{{x}^{2}}+{{y}^{2}}}=8+2y[/latex]

[latex]3\sqrt{{{x}^{2}}+{{y}^{2}}}=4+y[/latex]

[latex]9\left( {{x}^{2}}+{{y}^{2}} \right)=16+{{y}^{2}}+8y[/latex]

[latex]9{{x}^{2}}+8{{y}^{2}}-8y=16[/latex]

24. Let the equation of a line is [latex]3x+5y=15[/latex] and a point P on this line is equidistant from x and y axis.

In which quadrant that P lies.

(A) 1st (B) 3rd (C) 4th (D) none of these

Solution:

[latex]3x+5y=15[/latex]

[latex]\frac{x}{5}+\frac{y}{3}=1[/latex]

If [latex]y=x\Rightarrow 8x=15[/latex]

[latex]\Rightarrow x=\frac{15}{8},y=\frac{15}{8}[/latex]

If [latex]y=-x\Rightarrow -2x=15[/latex]

[latex]\Rightarrow x=\frac{-15}{2},y=\frac{15}{2}[/latex]

Intersection of [latex]3x+5y=15[/latex] with [latex]y=x[/latex] and [latex]y=-x[/latex] will give required P.

[latex]\therefore \,\,\,P=\left( \frac{15}{8},\frac{15}{8} \right)[/latex] or [latex]P=\left( \frac{-15}{2},\frac{15}

{2} \right)[/latex]

[latex]\therefore[/latex] P lies in either 1st or 2nd quadrant.

[latex]\therefore[/latex] In the given options 2nd quadrant not given

[latex]\therefore[/latex] 1st quadrant is the answer.

25. The perpendicular distance of point [latex]\left( 2,-1,4 \right)[/latex] from the line [latex]\frac{x+3}{10}=\

frac{y-2}{-7}=\frac{z}{1}[/latex] lies between 13 BYJU’s Classes4th Floor, Prince Kushal Towers, Mount Road, Chennai -02. PH: 9289000333

Page 14: cdn1.byjus.com · Web view[latex]A=\left[ \begin{matrix} \cos \alpha & -\sin \alpha \\ \sin \alpha & \cos \alpha \\ \end{matrix} \right]\Rightarrow {{A}^{2}}\left[ \begin{matrix}

MATHS – JEE MAINS – SHIFT – I

(A) (2, 3) (B) (3, 4) (C) (4, 5) (D) (1, 2)

Solution:

Let P be (2, -1, 4)

Let Q is a point on line [latex]\frac{x+3}{10}=\frac{y-2}{-7}=\frac{z}{1}=\lambda[/latex]

i.e. [latex]Q=\left( 10\lambda -3,-7\lambda +2,\lambda \right)[/latex]

Dr’s of PQ [latex]=\left( {{x}_{2}}-{{x}_{1}},{{y}_{2}}-{{y}_{1}},{{z}_{2}}-{{z}_{1}} \right)[/latex]

[latex]=\left( 10\lambda -5,-7\lambda +3,\lambda -4 \right)[/latex]

[latex]{{a}_{1}}{{a}_{2}}+{{b}_{1}}{{b}_{2}}+{{c}_{1}}{{c}_{2}}=0[/latex]

[latex]\Rightarrow 10\left( 10\lambda -5 \right)-7\left( -7\lambda +3 \right)+1\left( \lambda -4 \

right)=0[/latex]

[latex]\Rightarrow 100\lambda -50+49\lambda -21+\lambda -4=0[/latex]

[latex]\Rightarrow 150\lambda -75=0\Rightarrow \lambda =\frac{1}{2}[/latex]

[latex]PQ=\sqrt{{{\left( 10\lambda -5 \right)}^{2}}+{{\left( -7\lambda +3 \right)}^{2}}+{{\left( \lambda -4 \

right)}^{2}}}[/latex]

[latex]\sqrt{0+{{\left( \frac{-7}{2}+3 \right)}^{2}}+{{\left( \frac{1}{2}-4 \right)}^{2}}}=\sqrt{\

frac{50}{4}}=\sqrt{12.5}[/latex]

[latex]\sqrt{12.5}[/latex] lies between (3, 4)

26. If a plane passes through intersection of planes [latex]2x-y-4=0[/latex] and [latex]y+2z-4=0[/latex] and also

passes through the point (1, 1, 0). Then the equation of the plane is

(A) [latex]x-y-z=0[/latex] (B) [latex]2x-z=0[/latex] (C) [latex]x+2z-1=0[/latex] (D)

[latex]x-z-1=0[/latex]

Solution:

Plane passing through intersection two planes is

[latex]{{P}_{1}}+\lambda {{P}_{2}}=0[/latex]

[latex]\left( 2x-y-4 \right)+\lambda \left( y+2z-4 \right)=0[/latex] … (1)

14 BYJU’s Classes4th Floor, Prince Kushal Towers, Mount Road, Chennai -02. PH: 9289000333

Page 15: cdn1.byjus.com · Web view[latex]A=\left[ \begin{matrix} \cos \alpha & -\sin \alpha \\ \sin \alpha & \cos \alpha \\ \end{matrix} \right]\Rightarrow {{A}^{2}}\left[ \begin{matrix}

MATHS – JEE MAINS – SHIFT – I

(1) passing through (1, 1, 0) [latex]\Rightarrow \left( 2-1-4 \right)+\lambda \left( 1-4 \right)=0[/latex]

[latex]\Rightarrow \lambda =-1[/latex]

[latex]\therefore[/latex] equation of plane is [latex]\left( 2x-y-4 \right)-1\left( y+2z-4 \right)=0[/latex]

[latex]\Rightarrow 2x+2y-2z=0[/latex]

[latex]\Rightarrow x+y-z=0[/latex]

27. If [latex]\left| \sqrt{x}-2 \right|+\sqrt{x}\left( \sqrt{x}-4 \right)=2[/latex] then sum of roots of equation is

(A) 12 (B) 8 (C) 4 (D) 16

Solution:

[latex]\left| \sqrt{x}-2 \right|+x-4\sqrt{x}=2[/latex]

[latex]\left| \sqrt{x}-2 \right|+{{\left( \sqrt{x} \right)}^{2}}-4\sqrt{x}+4=2+4[/latex]

[latex]\left| \sqrt{x}-2 \right|+{{\left( \sqrt{x}-2 \right)}^{2}}=6[/latex]

Let [latex]\left| \sqrt{x}-2 \right|=P[/latex]

[latex]{{P}^{2}}+P-6=0\,\,\Rightarrow {{P}^{2}}+3P-2P-6=0[/latex]

[latex]\Rightarrow p=-3[/latex] or [latex]P=2[/latex]

[latex]\left| \sqrt{x}-2 \right|=-3[/latex] which is not possible

[latex]\left| \sqrt{x}-2 \right|=2[/latex]

[latex]\sqrt{x}-2=\pm 2[/latex]

[latex]\sqrt{x}=2+2[/latex] or [latex]-2+2[/latex]

[latex]\sqrt{x}=4[/latex] or 0

[latex]x=16[/latex] or 0

[latex]\therefore[/latex] sum of roots [latex]=16+0=16[/latex]

28. [latex]4{{x}^{2}}+{{y}^{2}}=8,[/latex] Tangent at (1, 2) and another tangent at (a, b) are perpendicular then

[latex]{{a}^{2}}=[/latex]

15 BYJU’s Classes4th Floor, Prince Kushal Towers, Mount Road, Chennai -02. PH: 9289000333

Page 16: cdn1.byjus.com · Web view[latex]A=\left[ \begin{matrix} \cos \alpha & -\sin \alpha \\ \sin \alpha & \cos \alpha \\ \end{matrix} \right]\Rightarrow {{A}^{2}}\left[ \begin{matrix}

MATHS – JEE MAINS – SHIFT – I

(A) [latex]\frac{2}{17}[/latex] (B) [latex]\frac{1}{17}[/latex] (C) [latex]\

frac{8}{17}[/latex] (D) [latex]\frac{4}{17}[/latex]

Solution:

[latex]4{{x}^{2}}+{{y}^{2}}=8[/latex]

[latex]\frac{{{x}^{2}}}{2}+\frac{{{y}^{2}}}{8}=1\Rightarrow \frac{{{x}^{2}}}{{{\left( \sqrt{2} \

right)}^{2}}}+\frac{{{y}^{2}}}{{{\left( 2\sqrt{2} \right)}^{2}}}=1[/latex]

Let (a, b) [latex]=\left( \sqrt{2}\cos \theta ,2\sqrt{2}\sin \theta \right)[/latex]

Tangent at (1, 2) is

[latex]4x+2y=8\Rightarrow 2x+y=4\Rightarrow {{m}_{1}}=-2[/latex]

Tangent at [latex]\left( \sqrt{2}\cos \theta ,2\sqrt{2}\sin \theta \right)[/latex] is

[latex]4\sqrt{2}\cos \theta x+\left( 2\sqrt{2}\sin \theta \right)=y=8\Rightarrow 4\sqrt{2}\cos \theta +2\

sqrt{2}\sin \theta y=8[/latex]

[latex]\Rightarrow {{m}_{2}}=-2\cot \theta[/latex].

[latex]{{m}_{1}}{{m}_{2}}=-1[/latex]

[latex]4\cot \theta =-1\Rightarrow Tan\theta =-4[/latex]

[latex]\Rightarrow {{\cos }^{2}}\theta =\frac{1}{17}[/latex]

[latex]\Rightarrow \cos \theta =\pm \frac{1}{\sqrt{17}}[/latex]

[latex]a=\sqrt{2}\cos \theta[/latex]

[latex]{{a}^{2}}=2{{\cos }^{2}}\theta \Rightarrow {{a}^{2}}=\frac{2}{17}[/latex]

29. Find the magnitude of projection of vector [latex]2i+3j+k[/latex] on a vector which is perpendicular to the plane

containing vectors [latex]i+j+k[/latex] and [latex]i+2j+3k[/latex]

(A) [latex]\frac{\sqrt{3}}{\sqrt{2}}[/latex] (B) [latex]\frac{\sqrt{2}}{\sqrt{3}}[/latex]

(C) [latex]\frac{4}{\sqrt{3}}[/latex] (D) [latex]\frac{2\sqrt{2}}{\sqrt{3}}[/latex]

Solution:

Normal vector to the plane containing [latex]i+j+k[/latex] and [latex]i+2j+3k[/latex]is

16 BYJU’s Classes4th Floor, Prince Kushal Towers, Mount Road, Chennai -02. PH: 9289000333

Page 17: cdn1.byjus.com · Web view[latex]A=\left[ \begin{matrix} \cos \alpha & -\sin \alpha \\ \sin \alpha & \cos \alpha \\ \end{matrix} \right]\Rightarrow {{A}^{2}}\left[ \begin{matrix}

MATHS – JEE MAINS – SHIFT – I

[latex]\overrightarrow{n\,}=\left( i+j+k \right)\times \left( i+2j+3k \right)[/latex]

[latex]\overrightarrow{n\,}=i-2j+k[/latex]

Projection of [latex]\left( 2i+3j+k \right)[/latex] on [latex]\overrightarrow{\,n\,}[/latex]

[latex]=\left| \frac{\left( 2i+3j+k \right).\left( i-2j+k \right)}{\sqrt{1+4+1}} \right|=\frac{\left|

2-6+1 \right|}{\sqrt{6}}=\frac{3}{\sqrt{6}}=\frac{\sqrt{3}}{\sqrt{2}}[/latex]

30. If [latex]\alpha ={{\cos }^{-1}}\left( \frac{3}{5} \right)[/latex] and [latex]\beta =Ta{{n}^{-1}}\left( \frac{5}

{12} \right)[/latex] then [latex]\alpha -\beta =[/latex]

(A) [latex]Ta{{n}^{-1}}\left( \frac{33}{56} \right)[/latex] (B) [latex]Ta{{n}^{-1}}\left( \frac{31}{66} \

right)[/latex] (C) [latex]Ta{{n}^{-1}}\left( \frac{11}{56} \right)[/latex] (D) [latex]Ta{{n}^{-1}}\

left( \frac{9}{56} \right)[/latex]

Solution:

[latex]\alpha =Ta{{n}^{-1}}\frac{4}{3}[/latex]

[latex]\beta =Ta{{n}^{-1}}\frac{5}{12}[/latex]

[latex]\alpha -\beta =Ta{{n}^{-1}}\frac{4}{3}-Ta{{n}^{-1}}\frac{5}{12}=Ta{{n}^{-1}}\left( \frac{\frac{4}

{3}-\frac{5}{12}}{1+\frac{4}{3}.\frac{5}{12}} \right)[/latex]

[latex]=Ta{{n}^{-1}}\left( \frac{16-5}{36+20}.\frac{3}{8} \right)[/latex]

[latex]=Ta{{n}^{-1}}\left( \frac{33}{56} \right)[/latex]

17 BYJU’s Classes4th Floor, Prince Kushal Towers, Mount Road, Chennai -02. PH: 9289000333